A researcher discovered

This topic has expert replies
User avatar
Master | Next Rank: 500 Posts
Posts: 421
Joined: Sun Apr 17, 2011 4:27 am
Location: India
Thanked: 6 times
Followed by:2 members
GMAT Score:620

A researcher discovered

by vinni.k » Fri Jan 13, 2012 8:58 am
A researcher discovered that people who have low levels of immune-system activity tend to score much lower on tests of mental health than do people with normal or high immune-system activity. The researcher concluded from this experiment that the immune system protects against mental illness as well as against physical disease.

The researcher's conclusion depends on which of the following assumptions?

A. High immune-system activity protects against mental illness better than normal immune-system activity does.
B. Mental illness is similar to physical disease in its effects on body systems.
C. People with high immune-system activity cannot develop mental illness.
D. Mental illness does not cause people's immune-system activity to decrease.
E. Psychological treatment of mental illness is not as effective as is medical treatment.

Answer is D

[spoiler]Not sure what is wrong with B. I read most of the forums but didn't find any clear explanation for B.

Conclusion is : immune system protects against mental illness as well as against physical disease.

The author talks about mental illness and didn't talk about "physical illness" except in the conclusion he did mention "physical illness".
So, one can think it as a missing assumption about "physical illness".

Immune system protects against mental illness and physical illness
Mental illness = physical illness.
[/spoiler]
I would love to discuss on this point.

Looking forward to your replies

Thanks & Regards
Vinni

Master | Next Rank: 500 Posts
Posts: 382
Joined: Thu Mar 31, 2011 5:47 pm
Thanked: 15 times

by ArunangsuSahu » Fri Jan 13, 2012 8:54 pm
This is kind of Cause and Effect statement...may not be purely

IMMUNE SYSTEM protects against MENTAL ILLNESS

IMMUNE SYSTEM protects against PHYSICAL ILLNESS

But if already existing MENTAL ILLNESS lowers immune system then the last line of the CR will fail. or the above two statments will fail

(D) is the answer

User avatar
Master | Next Rank: 500 Posts
Posts: 421
Joined: Sun Apr 17, 2011 4:27 am
Location: India
Thanked: 6 times
Followed by:2 members
GMAT Score:620

by vinni.k » Sat Jan 14, 2012 4:15 am
ArunangsuSahu wrote:This is kind of Cause and Effect statement...may not be purely

IMMUNE SYSTEM protects against MENTAL ILLNESS

IMMUNE SYSTEM protects against PHYSICAL ILLNESS

But if already existing MENTAL ILLNESS lowers immune system then the last line of the CR will fail. or the above two statments will fail

(D) is the answer
Thanks ArunangsuSahu for your reply.

I completely agree with you that after negating (D), MENTAL ILLNESS lowers immune system, conclusion will be shattered that is immune system protection against mental illness.

But (A) looks more like a missing premise about physical illness to me.
Nothing is mentioned about "physical illness" in the argument except in the conclusion.
Choice (A) looks like a contender to me.

What do you think about A ?

Would really appreciate if anyone could help me in understanding the way A can be eliminated.

Regards
Vinni

Senior | Next Rank: 100 Posts
Posts: 62
Joined: Fri Aug 12, 2011 12:37 am
Thanked: 3 times
Followed by:1 members

by sk8legend408 » Sat Jan 14, 2012 7:14 am
Assumption questions are a bit tricky and as Arunangsu alluded to, it is best to use the negating test.

Now to answer your question...A can be eliminated because the argument does not depend on this assumption. This may be true and is a good trap answer but the argument does not require this to be true.

Also in CR questions...look for reverals in cause and effect as answer choices. If the passage said that one caused the other, what may be a strengthener or an assumption is the opposite is not true whereas for a weakener the opposite would be true.

I hope this helped in answering your question.

Master | Next Rank: 500 Posts
Posts: 382
Joined: Thu Mar 31, 2011 5:47 pm
Thanked: 15 times

by ArunangsuSahu » Tue Jan 17, 2012 9:57 am
@Vinnil

There cannot be anything called "High Immune System"

It is either "Normal Immune System" or "lower than Normal"

So (A) is not correct

User avatar
GMAT Instructor
Posts: 88
Joined: Tue Jan 17, 2012 5:01 pm
Thanked: 54 times
Followed by:37 members

by chris@magoosh » Tue Jan 17, 2012 5:28 pm
In reference to answer choice (A), let's take it apart and see why it is not a valid contender.

A. High immune-system activity protects against mental illness better than normal immune-system activity does.


The argument contends that the immune system protects against mental health, and poor mental health is a
result of a compromised immune system. What the argument does not consider is that poor mental healthy affects
the immune system (D).

If (A) is true, it does not weaken the argument. Indeed, it is consistent with the researchers claim - normal
immune-system activity protects against mental illness (researcher's claim); a high-immune system
provides even better protection.

That is all (A) is saying. Therefore it in no way hurts the researcher's claim.

User avatar
Master | Next Rank: 500 Posts
Posts: 421
Joined: Sun Apr 17, 2011 4:27 am
Location: India
Thanked: 6 times
Followed by:2 members
GMAT Score:620

by vinni.k » Thu Jan 19, 2012 7:53 am
Can anyone please explain in detail what is wrong with B. In my previous post my whole explanation referred to B not A.
(A) not even talked about "physical disease". I typed (A) by mistake, but my explanation clearly referred to (B) by including word "physical disease".

Thanks & Regards
Vinni

User avatar
Master | Next Rank: 500 Posts
Posts: 154
Joined: Thu Aug 26, 2010 9:32 am
Location: Chicago,IL
Thanked: 46 times
Followed by:19 members
GMAT Score:760

by rkanthilal » Thu Jan 19, 2012 7:11 pm
P1: People who have low levels of immune-system activity tend to score much lower on tests of mental health than do people with normal or high immune-system activity.
C1: The immune system protects against mental illness as well as against physical disease.

The researcher discovered a correlation between low levels of immune-system activity and low scores on tests of mental health. The researcher concludes that "the immune system protects against mental illness". In other words, the researcher concludes that the low level of immune-system activity is the cause of the low scores on tests of mental health.

The researcher's conclusion depends on which of the following assumptions?

This question is about the difference between correlation and causation.
vinni.k wrote: The author talks about mental illness and didn't talk about "physical illness" except in the conclusion he did mention "physical illness".
So, one can think it as a missing assumption about "physical illness".
B. "Mental illness is similar to physical disease in its effects on body systems". INCORRECT. The conclusion is about the relationship between low immune-system activity and mental illness. "Physical disease" is not relevant to this argument. You can remove that part of the conclusion and it would not make any difference. For example,

P1: People who have low levels of immune-system activity tend to score much lower on tests of mental health than do people with normal or high immune-system activity.
C1: The immune system protects against mental illness.

The argument is still the same. The researcher is concluding that low immune-system activity is CAUSING the low test scores (i.e. mental illness). You do not need to assume anything about the similarities between mental illness and physical illness for this conclusion to hold. Even if mental illness affects the body in a totally different way than physical disease, it could still be true that "the immune system protects against mental illness". Since the conclusion holds when we negate this answer choice, it is not an assumption.

D. "Mental illness does not cause people's immune-system activity to decrease". CORRECT. The researcher basically concludes that the low level of immune-system activity is causing the low scores on tests of mental health. However, it is possible that it is the other way around (mental illness is causing the low immune-system activity).

For the researcher's conclusion to hold he must assume that the low immune-system activity is causing the low test scores. This answer choice addresses this assumption because it eliminates the possibility that mental illness is causing low immune-system activity.

User avatar
Master | Next Rank: 500 Posts
Posts: 421
Joined: Sun Apr 17, 2011 4:27 am
Location: India
Thanked: 6 times
Followed by:2 members
GMAT Score:620

by vinni.k » Fri Jan 20, 2012 2:42 am
Thank you so much Ramesh for this fantastic explanation. You are awesome.

God bless you.

Regards
Vinni

rkanthilal wrote:P1: People who have low levels of immune-system activity tend to score much lower on tests of mental health than do people with normal or high immune-system activity.
C1: The immune system protects against mental illness as well as against physical disease.

The researcher discovered a correlation between low levels of immune-system activity and low scores on tests of mental health. The researcher concludes that "the immune system protects against mental illness". In other words, the researcher concludes that the low level of immune-system activity is the cause of the low scores on tests of mental health.

The researcher's conclusion depends on which of the following assumptions?

This question is about the difference between correlation and causation.
vinni.k wrote: The author talks about mental illness and didn't talk about "physical illness" except in the conclusion he did mention "physical illness".
So, one can think it as a missing assumption about "physical illness".
B. "Mental illness is similar to physical disease in its effects on body systems". INCORRECT. The conclusion is about the relationship between low immune-system activity and mental illness. "Physical disease" is not relevant to this argument. You can remove that part of the conclusion and it would not make any difference. For example,

P1: People who have low levels of immune-system activity tend to score much lower on tests of mental health than do people with normal or high immune-system activity.
C1: The immune system protects against mental illness.

The argument is still the same. The researcher is concluding that low immune-system activity is CAUSING the low test scores (i.e. mental illness). You do not need to assume anything about the similarities between mental illness and physical illness for this conclusion to hold. Even if mental illness affects the body in a totally different way than physical disease, it could still be true that "the immune system protects against mental illness". Since the conclusion holds when we negate this answer choice, it is not an assumption.

D. "Mental illness does not cause people's immune-system activity to decrease". CORRECT. The researcher basically concludes that the low level of immune-system activity is causing the low scores on tests of mental health. However, it is possible that it is the other way around (mental illness is causing the low immune-system activity).

For the researcher's conclusion to hold he must assume that the low immune-system activity is causing the low test scores. This answer choice addresses this assumption because it eliminates the possibility that mental illness is causing low immune-system activity.

User avatar
Master | Next Rank: 500 Posts
Posts: 154
Joined: Thu Aug 26, 2010 9:32 am
Location: Chicago,IL
Thanked: 46 times
Followed by:19 members
GMAT Score:760

by rkanthilal » Fri Jan 20, 2012 5:50 pm
vinni.k wrote:Thank you so much Ramesh for this fantastic explanation. You are awesome.

God bless you.

Regards
Vinni
Vinni, you're welcome...

User avatar
Legendary Member
Posts: 626
Joined: Fri Dec 23, 2011 2:50 am
Location: Ahmedabad
Thanked: 31 times
Followed by:10 members

by ronnie1985 » Fri Jan 20, 2012 8:03 pm
(D) Cause and effect...assumption
Follow your passion, Success as perceived by others shall follow you

Legendary Member
Posts: 1404
Joined: Tue May 20, 2008 6:55 pm
Thanked: 18 times
Followed by:2 members

by tanviet » Fri Feb 10, 2012 12:08 am
chris@magoosh wrote:In reference to answer choice (A), let's take it apart and see why it is not a valid contender.

A. High immune-system activity protects against mental illness better than normal immune-system activity does.


The argument contends that the immune system protects against mental health, and poor mental health is a
result of a compromised immune system. What the argument does not consider is that poor mental healthy affects
the immune system (D).

If (A) is true, it does not weaken the argument. Indeed, it is consistent with the researchers claim - normal
immune-system activity protects against mental illness (researcher's claim); a high-immune system
provides even better protection.

That is all (A) is saying. Therefore it in no way hurts the researcher's claim.
I highly appreciate the expert posting.

I wish the experts post the process of reasoning/prephrasing which they do before they go to answer choices. I think that we need to know something close to the correct answer before going to the answer choices. I think the prephrasing process is key important to the success because to be able to prephrasing you have to understand/paraphrasing the argument alreay and we need TO HAVE SOMETHING before going to answer choices. I we have nothing before we go to answer choices, how we can realize what we want.

pls, experts, detail the process of your thinking before you go to the answer choices. we focus to much on analysing each answer choice.

Master | Next Rank: 500 Posts
Posts: 150
Joined: Sat Apr 28, 2012 11:19 pm
Thanked: 4 times

by mv12 » Sun Jul 08, 2012 12:24 am
D it is.

Junior | Next Rank: 30 Posts
Posts: 27
Joined: Thu Aug 18, 2011 3:04 am

by Practicegmat » Sun Jul 08, 2012 10:56 pm
I agree that (D) is the assumption.
But, in a way (c) is also an assumption.
How do I eliminate (C) from the contenders?